Une inégalité à montrer

nyadis
Modifié (May 2023) dans Arithmétique
Bonjour, j'aimerais comprendre l'inégalité suivante $$ \sum_{\substack{k, \ell \geqslant 1 \\(k \ell, q)=1}} \frac{1}{\sqrt{k \ell}} f\left(\frac{\pi k \ell}{q}\right) \ll \sqrt{q} \log q , $$ avec l'information que  $f(x) \ll 1 /(x+1)^2$, valable pour $x \geqslant 0$.

Réponses

  • JLapin
    Modifié (May 2023)
    Que signifie $<<$ ?
    Et qui est $q$ ?
  • noix de totos
    Modifié (May 2023)
    La notation de Vinogradov $\ll$ est synonyme de celle de Landau $O$. Quant à $q$, en arithmétique il représente le plus souvent un entier $q > 1$.

    Pour répondre à la question posée, la somme est certainement égale à
    $$\sum_{\substack{n \geqslant 1 \\ (n,q)=1}} \frac{1}{\sqrt n} f \left( \frac{\pi n}{q} \right) \sum_{d \mid n} 1 = \sum_{\substack{n \geqslant 1 \\ (n,q)=1}} \frac{\tau(n)}{\sqrt n} f \left( \frac{\pi n}{q} \right).$$
    On la découpe en deux : 
    $$\sum_{\substack{n \geqslant 1 \\ (n,q)=1}} \frac{\tau(n)}{\sqrt n} f \left( \frac{\pi n}{q} \right) = \bigg( \sum_{\substack{n \leqslant q \\ (n,q)=1}} + \sum_{\substack{n > q  \\ (n,q)=1}} \bigg) \frac{\tau(n)}{\sqrt n} f \left( \frac{\pi n}{q} \right).$$
    Dans la $1$ère somme, on utilise $f(\pi n/q) \ll 1$, alors que dans la seconde, on majore $f(\pi n /q) \ll (q/n)^2$, ce qui donne
    $$\sum_{\substack{n \geqslant 1 \\ (n,q)=1}} \frac{\tau(n)}{\sqrt n} f \left( \frac{\pi n}{q} \right) \ll \sum_{n \leqslant q} \frac{\tau(n)}{\sqrt n} + q^2 \sum_{n > q} \frac{\tau(n)}{n^{5/2}} \ll q^{1/2} \log q $$
    où l'on a fait une sommation partielle à la fin.
  • le terme $q^2 \sum_{n > q} \frac{\tau(n)}{n^{5/2}} \ll q$  et avec la sommation partielle $ \sum_{n \leqslant q} \frac{\tau(n)}{\sqrt n} \ll \sqrt{q} \log q$. donc on devrait plutôt avoir à la fin que le tout $ \ll q \log q $ non ?
  • On applique aussi une sommation partielle pour cette $2$nde somme : 
    \begin{align*}
       q^2 \sum_{n > q} \frac{\tau(n)}{n^{5/2}} &= q^2 \left( - \frac{1}{q^{5/2}} \sum_{n \leqslant q} \tau(n) + \frac{5}{2} \int_q^\infty \frac{1}{t^{7/2}} \left( \sum_{n \leqslant t} \tau(n) \right) \textrm{d}t \right) \\
       & \ll q^2 \left( q^{-3/2} \log q + \int_q^\infty \frac{\log t}{t^{5/2}} \textrm{d}t \right) \ll q^{2-3/2} \log q \ll q^{1/2} \log q 
    \end{align*}
    où l'on a utilisé la majoration classique $\sum_{n \leqslant t} \tau(n) \ll t \log t$.
Connectez-vous ou Inscrivez-vous pour répondre.